Menu Close

Category: Integration

let-F-x-x-2-x-3-sin-t-t-x-dt-1-calculate-lim-x-0-F-x-and-lim-x-F-x-2-calculste-lim-x-0-F-x-and-lim-x-F-x-

Question Number 63273 by mathmax by abdo last updated on 01/Jul/19 $${let}\:{F}\left({x}\right)\:=\int_{{x}^{\mathrm{2}} } ^{{x}^{\mathrm{3}} } \:\:\:\:\:\frac{{sin}\left({t}\right)}{{t}+{x}}\:{dt} \\ $$$$\left.\mathrm{1}\right)\:{calculate}\:{lim}_{{x}\rightarrow\mathrm{0}} \:{F}\left({x}\right)\:{and}\:{lim}_{{x}\rightarrow+\infty} {F}\left({x}\right) \\ $$$$\left.\mathrm{2}\right){calculste}\:{lim}_{{x}\rightarrow\mathrm{0}} \:{F}^{'} \left({x}\right)\:{and}\:{lim}_{{x}\rightarrow+\infty} \:{F}^{'}…

x-tan-x-dx-

Question Number 63261 by aliesam last updated on 01/Jul/19 $$\int{x}\:{tan}\left({x}\right)\:{dx} \\ $$ Commented by kaivan.ahmadi last updated on 01/Jul/19 $${u}={tanx}\Rightarrow{du}=\frac{{dx}}{\mathrm{1}+{x}^{\mathrm{2}} } \\ $$$${dv}={xdx}\Rightarrow{v}=\frac{{x}^{\mathrm{2}} }{\mathrm{2}} \\…

dx-1-x-2-1-x-2-

Question Number 128775 by bramlexs22 last updated on 10/Jan/21 $$\int\:\frac{\mathrm{dx}}{\left(\mathrm{1}−\mathrm{x}\right)^{\mathrm{2}} \:\sqrt{\mathrm{1}−\mathrm{x}^{\mathrm{2}} }}\:? \\ $$ Answered by liberty last updated on 10/Jan/21 $$\:\mathrm{let}\:\varphi\:=\:\sqrt{\frac{\mathrm{1}+\mathrm{x}}{\mathrm{1}−\mathrm{x}}}\:\Rightarrow\mathrm{x}\:=\:\frac{\varphi^{\mathrm{2}} −\mathrm{1}}{\varphi^{\mathrm{2}} +\mathrm{1}}\:\mathrm{and}\:\mathrm{dx}\:=\:\frac{\mathrm{4}\varphi}{\left(\varphi^{\mathrm{2}} +\mathrm{1}\right)^{\mathrm{2}}…

let-B-x-y-0-1-1-t-x-1-t-y-1-dt-1-study-the-convergence-of-B-x-y-1-prove-that-B-x-y-B-y-x-prove-that-B-x-y-0-t-x-1-1-t-x-y-dt-2-prove-that-B-x-y-x-y-

Question Number 63232 by mathmax by abdo last updated on 01/Jul/19 $${let}\:{B}\left({x},{y}\right)\:=\int_{\mathrm{0}} ^{\mathrm{1}} \left(\mathrm{1}−{t}\right)^{{x}−\mathrm{1}} {t}^{{y}−\mathrm{1}} \:{dt} \\ $$$$\left.\mathrm{1}\right)\:{study}\:{the}\:{convergence}\:{of}\:{B}\left({x},{y}\right) \\ $$$$\left.\mathrm{1}\right)\:{prove}\:{that}\:{B}\left({x},{y}\right)={B}\left({y},{x}\right) \\ $$$${prove}\:{that}\:{B}\left({x},{y}\right)\:=\int_{\mathrm{0}} ^{\infty} \:\:\frac{{t}^{{x}−\mathrm{1}} }{\left(\mathrm{1}+{t}\right)^{{x}+{y}}…

Given-a-function-f-satisfy-f-x-3f-x-If-1-2-f-x-dx-2-then-2-1-f-x-dx-

Question Number 128750 by bemath last updated on 10/Jan/21 $$\mathrm{Given}\:\mathrm{a}\:\mathrm{function}\:\mathrm{f}\:\mathrm{satisfy}\:\mathrm{f}\left(−\mathrm{x}\right)=\mathrm{3f}\left(\mathrm{x}\right). \\ $$$$\mathrm{If}\:\int_{−\mathrm{1}} ^{\:\mathrm{2}} \mathrm{f}\left(\mathrm{x}\right)\:\mathrm{dx}\:=\:\mathrm{2}\:\mathrm{then}\:\int_{−\mathrm{2}} ^{\:\mathrm{1}} \mathrm{f}\left(\mathrm{x}\right)\mathrm{dx}=? \\ $$ Commented by mr W last updated on…